Solving $ 2< x^2 -[x]<5$ The 2019 Stack Overflow Developer Survey Results Are In ...

Do warforged have souls?

What can I do if neighbor is blocking my solar panels intentionally?

Example of compact Riemannian manifold with only one geodesic.

How to determine omitted units in a publication

US Healthcare consultation for visitors

Didn't get enough time to take a Coding Test - what to do now?

Presidential Pardon

How to politely respond to generic emails requesting a PhD/job in my lab? Without wasting too much time

Does Parliament need to approve the new Brexit delay to 31 October 2019?

Homework question about an engine pulling a train

University's motivation for having tenure-track positions

Sub-subscripts in strings cause different spacings than subscripts

Can withdrawing asylum be illegal?

Can the DM override racial traits?

How do you keep chess fun when your opponent constantly beats you?

What to do when moving next to a bird sanctuary with a loosely-domesticated cat?

Accepted by European university, rejected by all American ones I applied to? Possible reasons?

Is every episode of "Where are my Pants?" identical?

How do I design a circuit to convert a 100 mV and 50 Hz sine wave to a square wave?

Was credit for the black hole image misappropriated?

If I score a critical hit on an 18 or higher, what are my chances of getting a critical hit if I roll 3d20?

What is the role of 'For' here?

Make it rain characters

What do I do when my TA workload is more than expected?



Solving $ 2



The 2019 Stack Overflow Developer Survey Results Are In
Announcing the arrival of Valued Associate #679: Cesar Manara
Planned maintenance scheduled April 17/18, 2019 at 00:00UTC (8:00pm US/Eastern)Integer solutions of nonhomogeneous linear inequalitiesHow to solve these inequalities?Extracting a function from set of inequalitiesSolve the following quadratic inequalities by graphing the corresponding functionHow to solve inequalities with one variable by number lineQuadratic formula in double inequalitiesHow can I prove these inequalities involving PGF?Equality involving greatest integerSolution of a system of quadratic inequalitiesAn inequality involving irrational numbers












4












$begingroup$


How to solve inequalities in which we have quadratic terms and greatest integer function.



$$ 2< x^2 -[x]<5$$.
[.] is greatest integer function.
Do we need to break into the cases as [0,1), [1,2) and so on?










share|cite|improve this question











$endgroup$












  • $begingroup$
    Yes, that is exactly what you have to do.
    $endgroup$
    – Kavi Rama Murthy
    1 hour ago
















4












$begingroup$


How to solve inequalities in which we have quadratic terms and greatest integer function.



$$ 2< x^2 -[x]<5$$.
[.] is greatest integer function.
Do we need to break into the cases as [0,1), [1,2) and so on?










share|cite|improve this question











$endgroup$












  • $begingroup$
    Yes, that is exactly what you have to do.
    $endgroup$
    – Kavi Rama Murthy
    1 hour ago














4












4








4


1



$begingroup$


How to solve inequalities in which we have quadratic terms and greatest integer function.



$$ 2< x^2 -[x]<5$$.
[.] is greatest integer function.
Do we need to break into the cases as [0,1), [1,2) and so on?










share|cite|improve this question











$endgroup$




How to solve inequalities in which we have quadratic terms and greatest integer function.



$$ 2< x^2 -[x]<5$$.
[.] is greatest integer function.
Do we need to break into the cases as [0,1), [1,2) and so on?







functions inequality ceiling-function






share|cite|improve this question















share|cite|improve this question













share|cite|improve this question




share|cite|improve this question








edited 1 min ago









YuiTo Cheng

2,4064937




2,4064937










asked 1 hour ago









mavericmaveric

91912




91912












  • $begingroup$
    Yes, that is exactly what you have to do.
    $endgroup$
    – Kavi Rama Murthy
    1 hour ago


















  • $begingroup$
    Yes, that is exactly what you have to do.
    $endgroup$
    – Kavi Rama Murthy
    1 hour ago
















$begingroup$
Yes, that is exactly what you have to do.
$endgroup$
– Kavi Rama Murthy
1 hour ago




$begingroup$
Yes, that is exactly what you have to do.
$endgroup$
– Kavi Rama Murthy
1 hour ago










2 Answers
2






active

oldest

votes


















5












$begingroup$

Hint: use $$x-1<[x]leq x$$ and then solve some quadratic inequalites like



$$ x^2-5<[x] implies x^2-5<ximplies x^2-x-6<0$$



so $xin (-2,3)$ and so on...




  • If $xin (-2,-1)$ then $[x]=-2 $ so $0<x^2<3$ so $-sqrt{3}<x<sqrt{3}$ so $xin(-sqrt{3},-1)$

  • If $xin [-1,0)$ ...






share|cite|improve this answer











$endgroup$





















    2












    $begingroup$

    While breaking up into integer intervals always works, sometimes it is too much work.



    Let's say you instead had $$
    213 < x^2 - lfloor x rfloor < 505 $$

    You really don't want to consider $24$ cases individually.



    So you get clever and change this to a pair of simultaneous inequalities: Let $x = n + y$ with $n in Bbb Z$ and $0 leq y < 1$. Then the inequalities are
    $$
    left{ begin{array}{l} 213 < (n+y)^2 - n < 505 \ 0 leq y < 1 end{array} right.
    $$

    Then $$213 < n^2 + (2y-1)n +y^2 , 0 leq y < 1 implies 213 < n^2 + (1)n + 1
    $$

    and this shows that $n geq 15$ (you can get that with one application of the quadratic formula), cutting out a lot of the work. Similarly,
    $$ n^2 + (2y-1)n +y^2 < 505 , 0 leq y < 1 implies n^2 + (-1)n + 0 < 505
    $$

    and this shows that $n < 23$.



    Finally, you can also justify only examining the allowed values of $y$ in the boundary cases ($n = 15$ and $n = 22$); in between, any value of $y$ works. (This might not be the case for cubic expressions, for example.)



    So for example, on the low end, you would need to solve for $y$ in
    $$
    213 < (15+y)^2 - 15 = 210 - 30 y + y^2 \
    y^2 -30 y -3 > 0\
    y > frac12( sqrt{912} -30 ) \
    x > (sqrt{228} - 15) + 15 implies x > sqrt{228}
    $$

    and similarly you need to consider the case of $n=22$ to get the upper limit for $x$.






    share|cite|improve this answer









    $endgroup$














      Your Answer








      StackExchange.ready(function() {
      var channelOptions = {
      tags: "".split(" "),
      id: "69"
      };
      initTagRenderer("".split(" "), "".split(" "), channelOptions);

      StackExchange.using("externalEditor", function() {
      // Have to fire editor after snippets, if snippets enabled
      if (StackExchange.settings.snippets.snippetsEnabled) {
      StackExchange.using("snippets", function() {
      createEditor();
      });
      }
      else {
      createEditor();
      }
      });

      function createEditor() {
      StackExchange.prepareEditor({
      heartbeatType: 'answer',
      autoActivateHeartbeat: false,
      convertImagesToLinks: true,
      noModals: true,
      showLowRepImageUploadWarning: true,
      reputationToPostImages: 10,
      bindNavPrevention: true,
      postfix: "",
      imageUploader: {
      brandingHtml: "Powered by u003ca class="icon-imgur-white" href="https://imgur.com/"u003eu003c/au003e",
      contentPolicyHtml: "User contributions licensed under u003ca href="https://creativecommons.org/licenses/by-sa/3.0/"u003ecc by-sa 3.0 with attribution requiredu003c/au003e u003ca href="https://stackoverflow.com/legal/content-policy"u003e(content policy)u003c/au003e",
      allowUrls: true
      },
      noCode: true, onDemand: true,
      discardSelector: ".discard-answer"
      ,immediatelyShowMarkdownHelp:true
      });


      }
      });














      draft saved

      draft discarded


















      StackExchange.ready(
      function () {
      StackExchange.openid.initPostLogin('.new-post-login', 'https%3a%2f%2fmath.stackexchange.com%2fquestions%2f3185867%2fsolving-2-x2-x5%23new-answer', 'question_page');
      }
      );

      Post as a guest















      Required, but never shown

























      2 Answers
      2






      active

      oldest

      votes








      2 Answers
      2






      active

      oldest

      votes









      active

      oldest

      votes






      active

      oldest

      votes









      5












      $begingroup$

      Hint: use $$x-1<[x]leq x$$ and then solve some quadratic inequalites like



      $$ x^2-5<[x] implies x^2-5<ximplies x^2-x-6<0$$



      so $xin (-2,3)$ and so on...




      • If $xin (-2,-1)$ then $[x]=-2 $ so $0<x^2<3$ so $-sqrt{3}<x<sqrt{3}$ so $xin(-sqrt{3},-1)$

      • If $xin [-1,0)$ ...






      share|cite|improve this answer











      $endgroup$


















        5












        $begingroup$

        Hint: use $$x-1<[x]leq x$$ and then solve some quadratic inequalites like



        $$ x^2-5<[x] implies x^2-5<ximplies x^2-x-6<0$$



        so $xin (-2,3)$ and so on...




        • If $xin (-2,-1)$ then $[x]=-2 $ so $0<x^2<3$ so $-sqrt{3}<x<sqrt{3}$ so $xin(-sqrt{3},-1)$

        • If $xin [-1,0)$ ...






        share|cite|improve this answer











        $endgroup$
















          5












          5








          5





          $begingroup$

          Hint: use $$x-1<[x]leq x$$ and then solve some quadratic inequalites like



          $$ x^2-5<[x] implies x^2-5<ximplies x^2-x-6<0$$



          so $xin (-2,3)$ and so on...




          • If $xin (-2,-1)$ then $[x]=-2 $ so $0<x^2<3$ so $-sqrt{3}<x<sqrt{3}$ so $xin(-sqrt{3},-1)$

          • If $xin [-1,0)$ ...






          share|cite|improve this answer











          $endgroup$



          Hint: use $$x-1<[x]leq x$$ and then solve some quadratic inequalites like



          $$ x^2-5<[x] implies x^2-5<ximplies x^2-x-6<0$$



          so $xin (-2,3)$ and so on...




          • If $xin (-2,-1)$ then $[x]=-2 $ so $0<x^2<3$ so $-sqrt{3}<x<sqrt{3}$ so $xin(-sqrt{3},-1)$

          • If $xin [-1,0)$ ...







          share|cite|improve this answer














          share|cite|improve this answer



          share|cite|improve this answer








          edited 24 mins ago

























          answered 33 mins ago









          Maria MazurMaria Mazur

          49.9k1361125




          49.9k1361125























              2












              $begingroup$

              While breaking up into integer intervals always works, sometimes it is too much work.



              Let's say you instead had $$
              213 < x^2 - lfloor x rfloor < 505 $$

              You really don't want to consider $24$ cases individually.



              So you get clever and change this to a pair of simultaneous inequalities: Let $x = n + y$ with $n in Bbb Z$ and $0 leq y < 1$. Then the inequalities are
              $$
              left{ begin{array}{l} 213 < (n+y)^2 - n < 505 \ 0 leq y < 1 end{array} right.
              $$

              Then $$213 < n^2 + (2y-1)n +y^2 , 0 leq y < 1 implies 213 < n^2 + (1)n + 1
              $$

              and this shows that $n geq 15$ (you can get that with one application of the quadratic formula), cutting out a lot of the work. Similarly,
              $$ n^2 + (2y-1)n +y^2 < 505 , 0 leq y < 1 implies n^2 + (-1)n + 0 < 505
              $$

              and this shows that $n < 23$.



              Finally, you can also justify only examining the allowed values of $y$ in the boundary cases ($n = 15$ and $n = 22$); in between, any value of $y$ works. (This might not be the case for cubic expressions, for example.)



              So for example, on the low end, you would need to solve for $y$ in
              $$
              213 < (15+y)^2 - 15 = 210 - 30 y + y^2 \
              y^2 -30 y -3 > 0\
              y > frac12( sqrt{912} -30 ) \
              x > (sqrt{228} - 15) + 15 implies x > sqrt{228}
              $$

              and similarly you need to consider the case of $n=22$ to get the upper limit for $x$.






              share|cite|improve this answer









              $endgroup$


















                2












                $begingroup$

                While breaking up into integer intervals always works, sometimes it is too much work.



                Let's say you instead had $$
                213 < x^2 - lfloor x rfloor < 505 $$

                You really don't want to consider $24$ cases individually.



                So you get clever and change this to a pair of simultaneous inequalities: Let $x = n + y$ with $n in Bbb Z$ and $0 leq y < 1$. Then the inequalities are
                $$
                left{ begin{array}{l} 213 < (n+y)^2 - n < 505 \ 0 leq y < 1 end{array} right.
                $$

                Then $$213 < n^2 + (2y-1)n +y^2 , 0 leq y < 1 implies 213 < n^2 + (1)n + 1
                $$

                and this shows that $n geq 15$ (you can get that with one application of the quadratic formula), cutting out a lot of the work. Similarly,
                $$ n^2 + (2y-1)n +y^2 < 505 , 0 leq y < 1 implies n^2 + (-1)n + 0 < 505
                $$

                and this shows that $n < 23$.



                Finally, you can also justify only examining the allowed values of $y$ in the boundary cases ($n = 15$ and $n = 22$); in between, any value of $y$ works. (This might not be the case for cubic expressions, for example.)



                So for example, on the low end, you would need to solve for $y$ in
                $$
                213 < (15+y)^2 - 15 = 210 - 30 y + y^2 \
                y^2 -30 y -3 > 0\
                y > frac12( sqrt{912} -30 ) \
                x > (sqrt{228} - 15) + 15 implies x > sqrt{228}
                $$

                and similarly you need to consider the case of $n=22$ to get the upper limit for $x$.






                share|cite|improve this answer









                $endgroup$
















                  2












                  2








                  2





                  $begingroup$

                  While breaking up into integer intervals always works, sometimes it is too much work.



                  Let's say you instead had $$
                  213 < x^2 - lfloor x rfloor < 505 $$

                  You really don't want to consider $24$ cases individually.



                  So you get clever and change this to a pair of simultaneous inequalities: Let $x = n + y$ with $n in Bbb Z$ and $0 leq y < 1$. Then the inequalities are
                  $$
                  left{ begin{array}{l} 213 < (n+y)^2 - n < 505 \ 0 leq y < 1 end{array} right.
                  $$

                  Then $$213 < n^2 + (2y-1)n +y^2 , 0 leq y < 1 implies 213 < n^2 + (1)n + 1
                  $$

                  and this shows that $n geq 15$ (you can get that with one application of the quadratic formula), cutting out a lot of the work. Similarly,
                  $$ n^2 + (2y-1)n +y^2 < 505 , 0 leq y < 1 implies n^2 + (-1)n + 0 < 505
                  $$

                  and this shows that $n < 23$.



                  Finally, you can also justify only examining the allowed values of $y$ in the boundary cases ($n = 15$ and $n = 22$); in between, any value of $y$ works. (This might not be the case for cubic expressions, for example.)



                  So for example, on the low end, you would need to solve for $y$ in
                  $$
                  213 < (15+y)^2 - 15 = 210 - 30 y + y^2 \
                  y^2 -30 y -3 > 0\
                  y > frac12( sqrt{912} -30 ) \
                  x > (sqrt{228} - 15) + 15 implies x > sqrt{228}
                  $$

                  and similarly you need to consider the case of $n=22$ to get the upper limit for $x$.






                  share|cite|improve this answer









                  $endgroup$



                  While breaking up into integer intervals always works, sometimes it is too much work.



                  Let's say you instead had $$
                  213 < x^2 - lfloor x rfloor < 505 $$

                  You really don't want to consider $24$ cases individually.



                  So you get clever and change this to a pair of simultaneous inequalities: Let $x = n + y$ with $n in Bbb Z$ and $0 leq y < 1$. Then the inequalities are
                  $$
                  left{ begin{array}{l} 213 < (n+y)^2 - n < 505 \ 0 leq y < 1 end{array} right.
                  $$

                  Then $$213 < n^2 + (2y-1)n +y^2 , 0 leq y < 1 implies 213 < n^2 + (1)n + 1
                  $$

                  and this shows that $n geq 15$ (you can get that with one application of the quadratic formula), cutting out a lot of the work. Similarly,
                  $$ n^2 + (2y-1)n +y^2 < 505 , 0 leq y < 1 implies n^2 + (-1)n + 0 < 505
                  $$

                  and this shows that $n < 23$.



                  Finally, you can also justify only examining the allowed values of $y$ in the boundary cases ($n = 15$ and $n = 22$); in between, any value of $y$ works. (This might not be the case for cubic expressions, for example.)



                  So for example, on the low end, you would need to solve for $y$ in
                  $$
                  213 < (15+y)^2 - 15 = 210 - 30 y + y^2 \
                  y^2 -30 y -3 > 0\
                  y > frac12( sqrt{912} -30 ) \
                  x > (sqrt{228} - 15) + 15 implies x > sqrt{228}
                  $$

                  and similarly you need to consider the case of $n=22$ to get the upper limit for $x$.







                  share|cite|improve this answer












                  share|cite|improve this answer



                  share|cite|improve this answer










                  answered 31 mins ago









                  Mark FischlerMark Fischler

                  34.1k12552




                  34.1k12552






























                      draft saved

                      draft discarded




















































                      Thanks for contributing an answer to Mathematics Stack Exchange!


                      • Please be sure to answer the question. Provide details and share your research!

                      But avoid



                      • Asking for help, clarification, or responding to other answers.

                      • Making statements based on opinion; back them up with references or personal experience.


                      Use MathJax to format equations. MathJax reference.


                      To learn more, see our tips on writing great answers.




                      draft saved


                      draft discarded














                      StackExchange.ready(
                      function () {
                      StackExchange.openid.initPostLogin('.new-post-login', 'https%3a%2f%2fmath.stackexchange.com%2fquestions%2f3185867%2fsolving-2-x2-x5%23new-answer', 'question_page');
                      }
                      );

                      Post as a guest















                      Required, but never shown





















































                      Required, but never shown














                      Required, but never shown












                      Required, but never shown







                      Required, but never shown

































                      Required, but never shown














                      Required, but never shown












                      Required, but never shown







                      Required, but never shown







                      Popular posts from this blog

                      ORA-01691 (unable to extend lob segment) even though my tablespace has AUTOEXTEND onORA-01692: unable to...

                      Always On Availability groups resolving state after failover - Remote harden of transaction...

                      Circunscripción electoral de Guipúzcoa Referencias Menú de navegaciónLas claves del sistema electoral en...